Chang wants to rent a boat and spend at most $44. the boat cost $8 per hour, and chang has a discount coupon for $4 off. what are the possible numbers of hours chain could rent the boat? Use "t" for number of hours .

Answers

Answer 1
8t-4 is less than or equal to 44

t=6
Answer 2

Answer:

6 hours.

Step-by-step explanation:

Here's my work.

Chang only wanted to spend $44, and the boat costs $8/hr. So, the variable would only be 8t, and you would put 44 on the other side.

Then, you'd put -4 on the side with 8t, because you can take off $4.

So, the setup would be;

44 = 8t - 4.

Chang Wants To Rent A Boat And Spend At Most $44. The Boat Cost $8 Per Hour, And Chang Has A Discount

Related Questions

1. What is the central measure found by adding all numbers in the data set and then dividing by the number of values
A. data
B. mean
C. median
D. mode​

Answers

Answer:

mean will be find in that method

hence, correct option is B



Please help I will award brainless to whoever helps please it’s urgent

Answers

Answer:

128 sq units :)

Step-by-step explanation:

hope this helps! have a great day :)

A ball is dropped from a height of 5m. After each bounce it rises to 35% of its previous height. After how many bounces does the ball reach a Height of less than 50cm?

Answers

Answer:

After the third bounce the ball reaches a height of less than 50cm.

Step-by-step explanation:

Geometric sequence:

In a geometric sequence, the quotient between consecutive terms is always the same, and it's called common ratio. The nth term of a geometric sequence is given by:

[tex]A_n = A_0(r)^{n}[/tex]

In which [tex]A_0[/tex] is the first term and r is the common ratio.

A ball is dropped from a height of 5m.

This means that [tex]A_0 = 5[/tex]

After each bounce it rises to 35% of its previous height.

This means that [tex]r = 0.35[/tex]

Thus

[tex]A_n = A_0(r)^{n}[/tex]

[tex]A_n = 5(0.35)^{n}[/tex]

After how many bounces does the ball reach a Height of less than 50cm?

50cm = 0.5m. This is n for which [tex]A_n = 0.5[/tex]. Thus

[tex]A_n = 5(0.35)^{n}[/tex]

[tex]0.5 = 5(0.35)^{n}[/tex]

[tex](0.35)^n = \frac{0.5}{5}[/tex]

[tex](0.35)^n = 0.1[/tex]

[tex]\log{(0.35)^n} = \log{0.1}[/tex]

[tex]n\log{0.35} = \log{0.1}[/tex]

[tex]n = \frac{\log{0.1}}{\log{0.35}}[/tex]

[tex]n = 2.19[/tex]

Rounding up:

After the third bounce the ball reaches a height of less than 50cm.

What expression is equal to COS B????

Answers

Answer:

A/C

Step-by-step explanation:

Since the sum of the angles in a triangle equals 180°, and angle C is 90°.

That means angles A and B add up to 90°, that is, they are complementary angles. Therefore the cosine of B equals the sine of A.

We saw on the last page that sin A was the opposite side over the hypotenuse, that is, a/c. Hence, cos B equals a/c.

Your welcome! :)

Y=-4+b
I need a answer fast and thank you

Answers

If we solve for Y then answer is..

Y=b-4

If we solve for B then answer is..

b=Y+4

the answer would be positive four

if x and 5x are supplementary angles, find them​

Answers

Answer:345678098321

Step-by-step explanation: I think so lol

Answer:

x+5x=180( adj.suppl.ls)

6x=180

6x÷6=180÷6

x=30

I need help asap pls pls pls...

Answers

Answer:

c)

Step-by-step explanation:

please don't trust ;-;

The answer to this question is C. The surface area of a sphere is S=4pi radius square. Where r=20cm so substitute it into the formula and you will get 1600pi cm^2

What is the simplified expression for -3(2x-y)+2y+2(x+y)
Answers:
8x+y
y-4x
7y-4x
-4x-y

Answers

Answer:

7y - 4x

Step-by-step explanation:

Given

- 3(2x - y) + 2y + 2(x + y) ← distribute parenthesis

= - 6x + 3y + 2y + 2x + 2y ← collect like terms

= 7y - 4x

Surface area of a cylinder

Answers

Answer:

A=2πrh+2πr2

Step-by-step explanation:youre welcome

Step-by-step explanation:

πr^2+2πrh

(r= radius, h= height)

Brian says that ABCD is a right triangle. Is he correct? (Round your answer to the nearest whole number) Explain.
17 m
21.7 m
С
D 13.5 m

Answers

21.7²=17²+13.5²

21.7²=289+182.25

21.7²=471.25

21.7=√471.25

21.7=21.7

22(to the nearest whole number)

Answer:  Yes, it is a right triangle

But only if you rounded everything to the nearest whole number.

Otherwise, it's a bit off (but fairly close).

==========================================================

Explanation:

If that was a right triangle, then a^2+b^2 = c^2 would be the case (pythagorean theorem). The a,b,c refers to the sides of the triangle. The 'c' is always the longest side, where 'a' and 'b' can be in any order you want.

We have

a = 13.5b = 17c = 21.7

Which leads to

a^2+b^2 = c^2

(13.5)^2+(17)^2 = (21.7)^2

182.25 + 289 = 470.89

471.25 = 470.89

We don't get the same thing on both sides, so we don't have a right triangle.

However, your teacher mentions to round the results to the nearest whole number. The 471.25 on the left side becomes 471, while the 470.89 becomes 471.

So while the equation 471.25 = 470.89 is definitely false, both sides are close enough that they round to 471 = 471 which is true.

In other words, this isn't a right triangle but it's close enough to one. Based on this rounding criteria, Brian is correct.

Find the opposite of each polynomial.

Answers

Answer:

Step-by-step explanation:

The word "opposite" is indicated by the - sign out front. Mathematically, we multiply a -1 into the parenthesis which, not coincidentally, changes the signs of everything inside the parenthesis, making them "opposite". The opposite of this polynomial is

- a - b - c

find the surface area. 3in. 2in. 6in. 2in. 4in.​

Answers

Answer:

I think it's 4in is my answer

Algebra evaluate each expression if a = -3 ,b= - 4 , and c = 5

what is 1. 5bc

Answers

-30 is the answer to your question

20 Points!!!

A ping pong ball is released from a height of 60 centimeters (cm) and bounces to a height that is 3/4 the previous height. What function estimates the height, H, in cm of the ping pong ball after x bounces?

Enter a number in each empty box to correctly complete the function.

H = ____ (____)^x

Answers

Multiply the starting height (60) by the height of each bounce (3/4) raised to the number of bounces (x)

H = (60)(3/4)^x

The answer to the given expression is:

H = 60(3/4)^x

What do we mean by exponents?

A number is raised by another number which is called the power/exponent which implies that the number is multiplied by itself for that many number of times.

How do we solve for H?

After every release, it bounces 3/4th of the previously released height.

After 1st bounce H = 60(3/4)

After 2nd bounce, it will be 3/4th of the previous H,

∴ H = 60(3/4)(3/4) = 60(3/4)^2

Similarly, after the 3rd bounce, it will be 3/4th of the previous H,

∴ H = 60(3/4)^2 * (3/4) = 60(3/4)^3

So it goes on like this. After x bounces,

H = 60(3/4)^x

Learn more about the Exponents at

https://brainly.com/question/11464095

#SPJ2

A rectangle has an area of 24 square meters. The width of the rectangle is 4 meters. What is the length of the rectangle? meters

Answers

Answer:

6 meters

Step-by-step explanation:

Use the area formula, A = lw, where l is the length and w is the width.

Plug in 24 as the area and 4 as the width, then solve for the length (l):

A = lw

24 = l(4)

Divide each side by 4:

6 = l

So, the length of the rectangle is 6 meters

Answer:

6 m

Step-by-step explanation:

Area = width x length

Area= 24m²

Width= 4m

A= wl

24= 4 x l

[tex]\frac{24}{4}[/tex] = l

6m = l

Simplify (2 − + 1) + 5 and find its value for a = -2.

Answers

Answer:

Part A

a·(a² - a + 1) + 5·a = a³ - a² + 6·a

Part B

The value of a·(a² - a + 1) + 5·a, for a = -2 is -24

Step-by-step explanation:

Part A

The given function is a·(a² - a + 1) + 5·a

The function is simplified by expanding the product of sums into sums of products, as follows;

a·(a² - a + 1) + 5·a = a × a² - a × a + a × 1 + 5·a = a³ - a² + a + 5·a = a³ - a² + 6·a

The function, a·(a² - a + 1) + 5·a, in simplified format is therefore;

a·(a² - a + 1) + 5·a = a³ - a² + 6·a

Part B

When a = -2, we get;

(a³ - a² + 6·a)[tex]_{a = 2}[/tex] = (-2)³ - (-2)² + 6·(-2) = -8 - 4 - 12 = -24

solve by factoring
please help this is timed :(

Answers

you find values of 28 that add to -11! in this case thats -7 and -4. after that just plug in (x-7)(x-4)=0 so: x=4, x=7

Answer: X1= 4, X2= 7

Step-by-step explanation:

Please help me that is the volume???

Answers

Let us first find the hieght of the triangle:

= 16² = p² + 8²

= 256 = p² + 64

= 256 - 64 = p²

= √192 = p

= 13.85 = p

Area of equilateral trianglular prism:

= ⅓× l² × h

Here, l = length of side and h is height

= ⅓ × 16² × 13.85

= 256 × 13.85 × ⅓

= 1181.86 cm³

= 1182 cm³ (approx)

What’s is the answer plzzzz help

eacg fruit basket has 5 apples and 3 pears . what is the ratio of the total number of fruits to the numbers of pears in each fruit basket ?

Answers

Answer:

The ratio is 5:3 because there are 5apples and 3 pears

The ratio is 5:3 because there are 5apples and 3 рears

Drag the tiles to the correct boxes to complete the pair.

Match the pairs of values of f(x) and g(x) with the corresponding values of h(x) if h(x)= f(x)/g(x).

f(x) = x2 - 9, and g(x) = x - 3

f(x) = x2 - 4x + 3, and g(x)= x - 3

f(x) = x2 + 4x - 5, and g(x) = x - 1

f(x) = x2 - 16, and g(x) = x - 4

1.h(x) = x + 5—>
2.h(x) = x + 3—>
3.h(x) = x+4—>
4.h(x) = x-1—>

Answers

Answer:

1. h(x) = x + 5—> f(x) = x² + 4x - 5, and g(x) = x - 1

2. h(x) = x + 3 —> f(x) = x² - 9, and g(x) = x - 3

3. h(x) = x + 4—> f(x) = x² - 16, and g(x) = x - 4

4.h(x) = x - 1—> f(x) = x² - 4x + 3, and g(x)= x - 3

Step-by-step explanation:

A) f(x) = x² - 9, and g(x) = x - 3

We are told that; h(x) = f(x)/g(x)

f(x) = x² - 9 can be factorized to;

f(x) = (x + 3)(x - 3)

Thus; h(x) = (x + 3)(x - 3)/(x - 3)

(x - 3) will cancel out to give;

h(x) = x + 3

B) f(x) = x² - 4x + 3, and g(x)= x - 3

x² - 4x + 3 can be factorized as;

(x - 1)(x - 3)

Thus; f(x) = (x - 1)(x - 3)

h(x) = (x - 1)(x - 3)/(x - 3)

h(x) = x - 1

C) f(x) = x² + 4x - 5, and g(x) = x - 1

x² + 4x - 5 can be factorized as;

(x - 1)(x + 5)

Thus; f(x) = (x - 1)(x + 5)

h(x) = (x - 1)(x + 5)/(x - 1)

h(x) = x + 5

D) f(x) = x² - 16, and g(x) = x - 4

x² - 16 can be expressed as;

(x + 4)(x - 4)

Thus; f(x) = (x + 4)(x - 4)

h(x) = (x + 4)(x - 4)/(x - 4)

h(x) = x + 4

I need help............​

Answers

Problem 39

Because AB is parallel to DC, this means the consecutive interior angles B and C are supplementary (these angles are adjacent to either parallel line). They add to 180 degrees.

B+C = 180

107+C = 180

C = 180-107

C = 73

Answer: 73 degrees

====================================================

Problem 40

We use the same idea from earlier. This time we have two pairs of parallel lines instead of one pair. Adjacent angles in any parallelogram always add to 180.

R+I = 180

If R = 70, then angle I = 180-R = 180-70 = 110

If angle I = 110, then angle N = 70 (because I+N = 180)

If angle N = 70, then angle G = 110 (because N+G = 180)

As you can see, opposite angles of a parallelogram are always congruent.

Answer:angle I = 110angle N = 70angle G = 110

Six kids and two adults are going to the circus. Kid's tickets are on sale for only half the price of adult tickets. The total cost is $48. How much is one kids ticket? -how much is one adult ticket

Answers

Answer:

One kids ticket is $4.80 and one adult ticket is $9.60

Step-by-step explanation:

Create a system of equations where k is the cost of a kids ticket and a is the cost of an adult ticket:

6k + 2a = 48

k = 1/2a

Solve by substitution by substituting the second equation into the first one:

6k + 2a = 48

6(1/2a) + 2a = 48

Simplify and solve for a:

3a + 2a = 48

5a = 48

a = 9.6

Find the cost of a kids ticket by dividing this by 2, since they are on sale for half the price of adult tickets.

9.6/2

= 4.8

One kids ticket is $4.80 and one adult ticket is $9.60

is How many oranges did
Sally eat last month? a statistical question

Answers

Answer:

25

Step-by-step explanation:

Lets say it's a 30 day month. Sally really likes oranges so she eats them a lot.

Which of the following has the largest y-intercept?

Answers

Answer:

I would say 2 because if you go by absolute value it is the largest.

Luis is building a new deck and needs to have a slab in of concrete poured.he knows the contractor charges an initial coast of $75 plus an additional $2.50 per square foot of concrete.which equation can be used to determine the coast (y),in dollars,to pour a concrete slab with an area of x square feet.

Answers

Answer:

[tex]y = 2.5x + 75[/tex]

Step-by-step explanation:

Write the equation of the line that passes through the points (-6,5) and (3,−5). Put your answer in fully reduced point-slope form, unless it is a vertical or horizontal line.

Answers

Answer:

[tex]\displaystyle y-5=-\frac{10}{9}(x+6)[/tex]

Or:

[tex]\displaystyle y+5=-\frac{10}{9}(x-3)[/tex]

Step-by-step explanation:

We want to write the equation of a line that passes through the points (-6, 5) and (3, -5) in point-slope form.

Point-slope form is given by:

[tex]y-y_1=m(x-x_1)[/tex]

Thus, first, we need to find the slope. We can use the slope formula:

[tex]\displaystyle m=\frac{\Delta y}{\Delta x}=\frac{(-5)-(5)}{(3)-(-6)}=\frac{-10}{9}=-\frac{10}{9}[/tex]

Next, we can use either of the two given points. I'll use (-6, 5). So, let (-6, 5) be (x₁, y₁). Substitute:

[tex]\displaystyle y-(5)=-\frac{10}{9}(x-(-6))[/tex]

Or, fully simplified:

[tex]\displaystyle y-5=\frac{-10}{9}(x+6)[/tex]

Using the other point, we will acquire:

[tex]\displaystyle y-(-5)=-\frac{10}{9}(x-(3))[/tex]

Or, simplified:

[tex]\displaystyle y+5=-\frac{10}{9}(x-3)[/tex]

Kara and Karl love steak, fried chicken, hamburgers, mashed potatoes, and french fries. They like green beans and peas. How many different meals including a meat, potatoes and a green vegetable can they make from these choices? 8 meals 16 meals a. C. b. 12 meals d. 24 meals​

Answers

Answer:

steak, chicken, and hamburger are meats. 3 meats.

The French fries and mashed potatoes are potatoes. 2 Potatoes.

And Green beans and peas are green vegetables. 2 Vegetables.

I think we just do 3 x 2 x 2 = 12 Meals.

hope that helps bby<3

Find the measure of each numbered angle.

Answers

Answer:

Angle 1 - 148

Angle 2 - 32

Angle 3 - 73

Angle 4 - 148

Angle 5 - 105

Step-by-step explanation:

Angle 1 forms a line with angle 32 and equals 180, therefore Angel 1 = 180 - 32 = 148

Angle 2 has the same value as the opposing angle, 32

Angle 3 combined with 32 and 75 equals 180, therefore

Angle 3 = 180 - 32 - 75

Angle 3 = 73

Angle 4 is equal to angle 1

Angle 5 combined with 43 and Angle 2 (32) equals 180

Angle 5 = 180 - 43 - 32

Angle 5 = 105

Which statement shows the correct value for the exponential
expression 34?
O 4x 4x4=64
O 3x4=12
3x3x3x3=81
O 3x3x3 = 27

Answers

Answer:

B. 3 x 3 x 3 x 3 = 81

Step-by-step explanation:

exponential expression 3⁴

This means multiplying 3 four times

= 3 × 3 × 3 × 3

= 81

B. 3 x 3 x 3 x 3 = 81 is the answer

Similarly, if you have 5³

This means you'll multiply 5 three times

5³ = 5 × 5 × 5

= 125

And

3¹ =

3 one time

3¹ = 3

Josh is trying to factor the expression
- 20a - 8 + 12b. He writes
-4 (5a + 2 + 3b)
a. What error did Josh likely make?
b. Factor the expression correctly.

Answers

Answer:

a) Josh likely made the negative and positive multiplication error.

b) -20a-8+12b

= -4(5a+2-3b)

Step-by-step explanation:

To check if you are correct, you can always simplify it again:

-4(5a+2-3b)

=-20a-8+12b

Remember:

Negative times negative equals to positive, negative times positive equals to negative!

I hope this is helpful! :)

Other Questions
Sarah uses 3/4 pound of blueberries to make 2/3 cup of jam. How many pounds of blueberries does she need to make one cup of jam help me please will give brainliest and 10 points YESSIR What is the horizontal distance between (5/3, -2) and (4, -2)?ANSWER QUICK GIVING BRAIN LEST An 11.5 mL portion of 0.162 M H3PO4 (aq) is to be titrated with 0.229 M NaOH (aq). What volume (in mL) of NaOH will it take to reach the equivalence point Which point on the x-axis lies on the line that passes point C and is parallel to line AB a) Two thirds of the class are freshmanIs it parameter or statistic The shortest side of an isosceles triangle is labeled, in inches, in the triangle below. The two longer sides are 6 inches longer than the shortest side. The perimeter of the triangle is 33 inches. Which statement is true about the graphed function?O Fx) < 0 over the interval (-0, 4)OFx) < 0 over the interval (-0, -3)O F(x) > 0 over the interval (-00, -3)O F(x) > 0 over the interval (-0, 4) What was an activity of the national war larbor board What property was used in step 1 to arrive at step 2?Step 1: 5 (x minus 7) = 55. Step 2: 5 x minus 35 = 55. Step 3: 5 x = 90. Step 4: x = 18.distributive propertyaddition property of equalitysubtraction property of equalitydivision property of equality PLEASE HURRY!What is the half-life of a 12 g sample of radioisotope that decayed to 6 g in 28years?A. 14 yearsB. 7 yearsC. 56 yearsD. 28 years wats the answer of 3/4 of 7/15 the current supplied by the battery is 0.05A calculate the current in each of the 180 resistors 100 points This system of inequalities models the scenario:2x + y 8x + y 4Part A: Describe the graph of the system of inequalities, including shading and the types of lines graphed. Provide a description of the solution set. pleaseeeee helppp meee!!! Ali surveyed his classmates to see who liked tennis. In all, 32 males liked tennis, and 6 males did not like tennis; 13 females liked tennis, and 4 females did not like tennis. How many females did Ali survey What kinds of details do we examine to learn more about the authors characterization within a text? magnesium hydroxide + sulfuric acid please Unit 3 Module 1 Tutorial: My Immediate Family (Edmentum/Plato Spanish 1 Semester 1) please help I need answers!!! Find the roots of the quadratic equation (4p-1) = 4p( Please show me how it works , Thankiuuu )